aileenann
Thanks Received: 227
Atticus Finch
Atticus Finch
 
Posts: 300
Joined: March 10th, 2009
 
This post thanked 1 time.
 
 

Q15 - Bowers: A few theorists hold

by aileenann Sat Mar 27, 2010 5:35 pm

This question says to determine which of the answer choices would most weaken the author’s argument. We’ll particularly focus on looking for something that directly weakens the conclusion, but we should keep in mind that weakening premises can also be an answer choice.

In this argument, the author concludes that any social philosophy that promotes chaos is a bad one, primarily because an acceptable philosophy must promote peace and order.
However, this should give you pause _ particularly the author’s substitution of the word chaos for anarchy. Initially the author defined anarchy as "the absence of government." While we don’t want to be overly demanding about how authors use words, now eliding this definition to pure "chaos" seems somewhat disingenuous.

A second problem - even accepting the author’s contention about the promotion of law and order _ is that the author also seems to assume that short-term "chaos" cannot lead to long term peace and order, or that chaos is necessarily the opposite of peace.

With these initial thoughts in mind, let’s look to the answer choices.

(A) describes precisely our concern above about chaos v. an absence of government. Let’s keep this for now.
(B) states that the argument is unconvincing. While this might be what we are aiming to show with a weakening question, this premise doesn’t give any specific reason to show that the argument is weak. For this reason, we should still stick with (A) over (B) in choosing a final answer.
(C) may be true in the "real world," but it has nothing to do with this argument and is in particular irrelevant because strength of numbers is not the reasoning the author relies upon.
(D) is wrong. It has two words we’ve seen in the argument _ "peace" and "flourish" _ but those words were not connected by the author in the way this answer choice suggests.
(E) may also be true in the "real world" like (C), but like (C) it insinuates something that the author simply did not do in the argument.

Hence our answer is (A). While (B) might be something that goes against the conclusion, it is extremely broad and lacks bite. (B) is nearly the equivalent of saying "The author is wrong." When evaluated from a purely logical standpoint, such an assertion should not be convincing by itself. You must, instead, provide a specific reason to weaken the text, as (A) does.

Have you noticed any other questions that have a tricky wrong answer like (B)?
User avatar
 
noah
Thanks Received: 1192
Atticus Finch
Atticus Finch
 
Posts: 1541
Joined: February 11th, 2009
 
This post thanked 5 times.
 
 

Re: Q15 Bowers: A few theorists hold

by noah Wed Sep 15, 2010 5:51 pm

Apparently, we love this argument, because we keep explaining it. Here's one I wrote:

This question can be quite tricky because it is wordy; but at it's core is what I call a "word shift".

The conclusion of the argument is that anarchy deserves no further attention. Why? Apparently, those theorists who contend that society could flourish in a condition of anarchy have ignored the fundamental principle of social philosophy: that an acceptable social order must promote peace and order and that any philosophy that countenances (sanctions) chaos deserves no further attention. This applies to anarchy, right? Since we have to find a flaw, we can expect that there's probably a gap or assumption. What's missing? If you haven't caught it yet, take a look at this basic diagram of the argument:

Premise: countenance chaos --> deserve no further attention.
Conclusion: anarchy --> deserves no further attention

We need to be sure that anarchy countenances chaos! We might think that ourselves (though I imagine some anarchists would disagree), but it needs to be established in the argument. Looking back, the argument refers to anarchy as "the absence of government." For this argument to be sound, we have to assume that that means that it promotes chaos. It's actually not so far-fetched to imagine someone arguing against this connection: Perhaps it's true (or at least arguable) that an absence of government promotes peace and order because it is governments that wage full scale war . . .

(A) notes this shift in meaning, from "absence of government" to "countenances chaos"

(B) is irrelevant. The reference to laissez-faire capitalism is mentioned as a rather tangential counter-premise; it is not necessary to de-bunk it as it is only offered as a philosophy that relates to anarchy and -- it seems to suggest -- that is accepted. For example, if you argue that lacrosse is a fun sport, noting that it is a great melding of soccer, football and hockey, does it weaken your argument for me to prove that those three sports are not fun? No it does not; perhaps the melding of the three sports is fun.

(C) is out of scope. There is no reference to how many people support anarchy.

(D) is out of scope. What does it mean to flourish in this argument?

(E) is unsupported. Bowers does not reject anarchy because it is extreme.
User avatar
 
noah
Thanks Received: 1192
Atticus Finch
Atticus Finch
 
Posts: 1541
Joined: February 11th, 2009
 
This post thanked 1 time.
 
 

Re: PT 59, S2, Q15 Bowers: A few theorists hold

by noah Wed Sep 15, 2010 5:53 pm

And, to drive home our love for this one, here's a third explanation!


It's a flaw question, so we should be on the look out for an assumption.

The argument's conclusion is that anarchy deserves no further attention. Why? Because it countenances chaos. And how do we know it countenances chaos? It's not stated as a fact -- instead, Bowers tells us that anarchy is "the absence of government." This seems like a big shift: from "absence of government" to "countenancing chaos." (A) describes this shift -- an illicit shift in meaning. Usually, this sort of answer -- "uses the word XXX in two different senses" -- is an easy elimination, as such a shift is generally obvious, but here it's tricky because we ourselves may assume that a lack of government means chaos. However, anarchists would probably disagree!

(B) is outside the scope of the core noted above.

(C) is irrelevant to the core as well -- number of people?

(D) the argument does not assume that any peaceful society will flourish, so there!

(E) the argument does not describe anarchism as extreme.
 
pinkdatura
Thanks Received: 1
Forum Guests
 
Posts: 55
Joined: September 26th, 2010
 
 
 

Re: PT 59, S2, Q15 Bowers: A few theorists hold

by pinkdatura Wed Sep 29, 2010 12:54 am

Thank you for the wonderful explanation, I still feel confused about the logic of this stimulus:
some anarchy societies could flourish, therefore they can be excused from the fundamental principle? The principle is acceptable society-->promote peace and order;
countenance chaos(=anarchy society)--> deserve no attention
~promote peace and order=countenance chaos
~deserve no attention=acceptable society

so for D, flourish, although mentioned in the stimulus, is not a major concern in argument, so out of scope?
Pls correct me...
 
cyruswhittaker
Thanks Received: 107
Forum Guests
 
Posts: 246
Joined: August 11th, 2010
 
 
trophy
Most Thanked
trophy
First Responder
 

Re: PT 59, S2, Q15 Bowers: A few theorists hold

by cyruswhittaker Thu Oct 07, 2010 7:06 pm

Can you check the following? ...

*Could another possible answer choice of this question have been, in order to point out the same flaw:

"the argument presumes, without justification, that the absence of government requires that it countenances chaos."

*OR, if it was a "necessary assumption" question, the answer choice might be stated as:

"it is not true that an absence of government never countenances chaos"

Just trying to get my thinking right for how they can change up the questions, choices, and terminology.
User avatar
 
noah
Thanks Received: 1192
Atticus Finch
Atticus Finch
 
Posts: 1541
Joined: February 11th, 2009
 
 
 

Re: PT 59, S2, Q15 Bowers: A few theorists hold

by noah Fri Oct 08, 2010 2:11 pm

pinkdatura Wrote:Thank you for the wonderful explanation, I still feel confused about the logic of this stimulus:
some anarchy societies could flourish, therefore they can be excused from the fundamental principle? The principle is acceptable society-->promote peace and order;
countenance chaos(=anarchy society)--> deserve no attention
~promote peace and order=countenance chaos
~deserve no attention=acceptable society

so for D, flourish, although mentioned in the stimulus, is not a major concern in argument, so out of scope?
Pls correct me...

I think you're over-doing the idea of notating every premise. The key is to boil down these argument to their core. The first clause of the first sentence, for example, is irrelevant.

(D) is incorrect because the argument simply does not assume that any peaceful society will flourish. This is combining various words in the argument, but not in any meaningful way. So, you're right, it's out of scope.
User avatar
 
noah
Thanks Received: 1192
Atticus Finch
Atticus Finch
 
Posts: 1541
Joined: February 11th, 2009
 
This post thanked 1 time.
 
 

Re: PT 59, S2, Q15 Bowers: A few theorists hold

by noah Fri Oct 08, 2010 2:13 pm

cyruswhittaker Wrote:Can you check the following? ...

*Could another possible answer choice of this question have been, in order to point out the same flaw:

"the argument presumes, without justification, that the absence of government requires that it countenances chaos."

*OR, if it was a "necessary assumption" question, the answer choice might be stated as:

"it is not true that an absence of government never countenances chaos"

Just trying to get my thinking right for how they can change up the questions, choices, and terminology.

I always feel I'm on shaky ground playing the what if game with the LSAT! But, these both seem on target. Nice thinking.
 
interestedintacos
Thanks Received: 58
Atticus Finch
Atticus Finch
 
Posts: 116
Joined: November 09th, 2010
 
 
trophy
Most Thanked
trophy
First Responder
 

Re: Q15 - Bowers: A few theorists hold

by interestedintacos Tue May 31, 2011 11:07 pm

My pre-phrase was that the argument begs the question. It says if A-->B, and then without justification claims that anarchy falls into A. But it's the very question of whether anarchy fits into A that's at stake! The theorists are saying society could flourish under anarchy, their assumption presumably that anarchy isn't necessarily leading to chaos. Merely assuming that anarchy fits into A is assuming the very conclusion Bowers is going for.

Bowers moves from an unbiased, objective definition of anarchy to using a loaded definition of anarchy--and if the second definition is correct then his argument is proven, but he merely assumes that definition without justification. So he's begging the question by loading the definition.
 
shirando21
Thanks Received: 16
Atticus Finch
Atticus Finch
 
Posts: 280
Joined: July 18th, 2012
 
 
 

Re: PT 59, S2, Q15 Bowers: A few theorists hold

by shirando21 Fri Nov 16, 2012 11:54 am

noah Wrote:And, to drive home our love for this one, here's a third explanation!


It's a flaw question, so we should be on the look out for an assumption.

The argument's conclusion is that anarchy deserves no further attention. Why? Because it countenances chaos. And how do we know it countenances chaos? It's not stated as a fact -- instead, Bowers tells us that anarchy is "the absence of government." This seems like a big shift: from "absence of government" to "countenancing chaos." (A) describes this shift -- an illicit shift in meaning. Usually, this sort of answer -- "uses the word XXX in two different senses" -- is an easy elimination, as such a shift is generally obvious, but here it's tricky because we ourselves may assume that a lack of government means chaos. However, anarchists would probably disagree!

(B) is outside the scope of the core noted above.

(C) is irrelevant to the core as well -- number of people?

(D) the argument does not assume that any peaceful society will flourish, so there!

(E) the argument does not describe anarchism as extreme.


yeah, I did assume that a lack of government means chaos... unfamiliar topic.

I found I missed a majority of flaw questions on PT59, which is not usual. It is probably because I am not comfortable with the topics of these arguments, what shall I do?
User avatar
 
noah
Thanks Received: 1192
Atticus Finch
Atticus Finch
 
Posts: 1541
Joined: February 11th, 2009
 
This post thanked 1 time.
 
 

Re: PT 59, S2, Q15 Bowers: A few theorists hold

by noah Fri Nov 16, 2012 3:52 pm

shirando21 Wrote:I found I missed a majority of flaw questions on PT59, which is not usual. It is probably because I am not comfortable with the topics of these arguments, what shall I do?

For review, go ahead and write out each one in core form, boiling down the content, and then boil it down to as essential a relationship as possible. For example:

anarchy is absence of government --> anarchy countenances chaos --> anarchy deserves no further attention

Then, anarchy is something --> so anarchy does something else --> so, ignore anarchy

and possibly: A is something --> so A must be something else --> so, we should think this about A

This is an effort to get your brain to grasp how gaps are formed, regardless of what the gap is about.

If you try this, tell me how it goes.
 
MayMay
Thanks Received: 0
Forum Guests
 
Posts: 25
Joined: January 02nd, 2013
 
 
 

Re: Q15 - Bowers: A few theorists hold

by MayMay Sat May 18, 2013 8:09 pm

first of all, thanks for the detailed explanations!

i do have a quick question, though--
for a recap, here is the bare bones of what we've established.

premise: acceptable social philosophy --> promote peace and order
premise: anything that leads to chaos --> ignore.
conclusion: ignore anarchy.

My problem is, however, i initially thought there was a third premise of anarchy = gives into chaos because of the "I.e."
having interpreted this third premise, i thought, oh ok. so anarchy is this chaotic thing so the conclusion does follow!
how did you know that the "social philosophy that countenances chaos i.e. anarchy" wasn't a premise?
User avatar
 
noah
Thanks Received: 1192
Atticus Finch
Atticus Finch
 
Posts: 1541
Joined: February 11th, 2009
 
This post thanked 1 time.
 
 

Re: Q15 - Bowers: A few theorists hold

by noah Mon May 20, 2013 12:07 pm

MayMay Wrote:first of all, thanks for the detailed explanations!

i do have a quick question, though--
for a recap, here is the bare bones of what we've established.

premise: acceptable social philosophy --> promote peace and order
premise: anything that leads to chaos --> ignore.
conclusion: ignore anarchy.

My problem is, however, i initially thought there was a third premise of anarchy = gives into chaos because of the "I.e."
having interpreted this third premise, i thought, oh ok. so anarchy is this chaotic thing so the conclusion does follow!
how did you know that the "social philosophy that countenances chaos i.e. anarchy" wasn't a premise?

If you look back at my explanation, I actually treated anarchy = chaos as an intermediate conclusion. The question is whether we can get from the first fact about anarchy--it's the absence of government--to this next statement--it's chaos. Seems fishy, and (A) exploits that.

You could think of the argument as this:

(Anarchy = lack of government --> Anarchy = chaos) + soc. phil. must promote peace --> Anarchy is not acceptable

As for why to not accept "chaos, i.e. anarchy"--the big reason is that we have some alleged support for it. Anytime support is given for something in an assumption family question, that connection is up for review!

Make sense?
 
mirroredshades
Thanks Received: 10
Forum Guests
 
Posts: 5
Joined: June 06th, 2013
 
This post thanked 2 times.
 
 

Re: Q15 - Bowers: A few theorists hold

by mirroredshades Fri Jun 07, 2013 7:15 am

I don't know if it'll help, but here's my two cents:

I loved this question. Easy read, easy answer. Maybe because I'm partial to the subject matter. Funny, because I also know it's generally unwise to bring in "outside" info to these LSAT questions, but as soon as I saw the word "anarchy" I thought to myself, "Gee, I wonder when the author is going to equate it somehow with chaos?" The thought dissipated halfway, but, hey, what do you know, he brought it up.

The author says we should stop paying attention to the view of anarchy. Why? It countenances (promotes, approves, supports, etc.) chaos. Whoa, where did that come from? I think anarchism would actually be more proper in terms of philosophical theorizing, but then it's so much easier to ignore all that and equate "condition of anarchy" with chaos, isn't it? :D

Anyway. The final sentence just seems so jarring, considering most of the rest of the stimulus seems relatively evenhanded. In the first sentence, the author states that the "extreme view" is that society could function in a state of anarchy, that is, the absence of government. In the final sentence, he says anarchy basically countenances chaos. The rest of the stimulus doesn't logically fly with these two sentences. Other than a flimsy "promote peace and order" tidbit, the author does nothing to come logically to his conclusion. He does not even make an effort to show that promoting absence of government promotes chaos, or that failing to promote peace and order promotes chaos.

I think even if you were reading quickly, you would see that the author subtly (well, maybe not) shifts the meaning of anarchy from absence of government to countenancing chaos. That's (A). I also think a keyword here is "illicitly": he goes from a relatively neutral view (him referring to it as an "extreme view" being arguable, of course, but that's irrelevant here) to full-blown promotion of chaos, with very little warrant.

Even if (A) seems less than convincing, the rest of the answers are even worse, and the job we have is to find the best one.

(B) Laissez-faire capitalism is mentioned almost in passing as an example of the theorists' "interesting arguments". The author never suggests that laissez-faire is a social philosophy and, that, even if it is or even if he did, never says we should reject it. He says we should reject anarchy.

(C) I appreciate the wisdom involved, but when I first read this answer I thought: "What in the world does this have to do with anything?" Answer: Nothing. Yes, I agree with the wisdom, but nowhere does the author suggest anything even remotely related.

(D) The author does not make any hints towards whether or not any society, one which is anarchic or one which is peaceful, etc., will flourish (or won't flourish). He states that some theorists think that society could in an absence of government, but never advances any position, himself. His argument is about advancing (or not advancing) social philosophies, not what will advance societies.

(E) The author does state that it is an "extreme view", but he is not saying we should reject the view because it can be described as extreme. He is saying we should reject the view because it countenances chaos.
"I have free will, but not of my own choice. I have never freely chosen to have free will. I have to have free will, whether I like it or not!"
-- Raymond Smullyan
User avatar
 
noah
Thanks Received: 1192
Atticus Finch
Atticus Finch
 
Posts: 1541
Joined: February 11th, 2009
 
 
 

Re: Q15 - Bowers: A few theorists hold

by noah Fri Jun 07, 2013 1:27 pm

mirroredshades Wrote:I don't know if it'll help, but here's my two cents:

I loved this question. Easy read, easy answer. Maybe because I'm partial to the subject matter. Funny, because I also know it's generally unwise to bring in "outside" info to these LSAT questions, but as soon as I saw the word "anarchy" I thought to myself, "Gee, I wonder when the author is going to equate it somehow with chaos?" The thought dissipated halfway, but, hey, what do you know, he brought it up.

The author says we should stop paying attention to the view of anarchy. Why? It countenances (promotes, approves, supports, etc.) chaos. Whoa, where did that come from? I think anarchism would actually be more proper in terms of philosophical theorizing, but then it's so much easier to ignore all that and equate "condition of anarchy" with chaos, isn't it? :D

Anyway. The final sentence just seems so jarring, considering most of the rest of the stimulus seems relatively evenhanded. In the first sentence, the author states that the "extreme view" is that society could function in a state of anarchy, that is, the absence of government. In the final sentence, he says anarchy basically countenances chaos. The rest of the stimulus doesn't logically fly with these two sentences. Other than a flimsy "promote peace and order" tidbit, the author does nothing to come logically to his conclusion. He does not even make an effort to show that promoting absence of government promotes chaos, or that failing to promote peace and order promotes chaos.

I think even if you were reading quickly, you would see that the author subtly (well, maybe not) shifts the meaning of anarchy from absence of government to countenancing chaos. That's (A). I also think a keyword here is "illicitly": he goes from a relatively neutral view (him referring to it as an "extreme view" being arguable, of course, but that's irrelevant here) to full-blown promotion of chaos, with very little warrant.

Even if (A) seems less than convincing, the rest of the answers are even worse, and the job we have is to find the best one.

(B) Laissez-faire capitalism is mentioned almost in passing as an example of the theorists' "interesting arguments". The author never suggests that laissez-faire is a social philosophy and, that, even if it is or even if he did, never says we should reject it. He says we should reject anarchy.

(C) I appreciate the wisdom involved, but when I first read this answer I thought: "What in the world does this have to do with anything?" Answer: Nothing. Yes, I agree with the wisdom, but nowhere does the author suggest anything even remotely related.

(D) The author does not make any hints towards whether or not any society, one which is anarchic or one which is peaceful, etc., will flourish (or won't flourish). He states that some theorists think that society could in an absence of government, but never advances any position, himself. His argument is about advancing (or not advancing) social philosophies, not what will advance societies.

(E) The author does state that it is an "extreme view", but he is not saying we should reject the view because it can be described as extreme. He is saying we should reject the view because it countenances chaos.

Thanks for the write-up. Funny that you had smelled the shift coming!
 
ganbayou
Thanks Received: 0
Atticus Finch
Atticus Finch
 
Posts: 213
Joined: June 13th, 2015
 
 
 

Re: Q15 - Bowers: A few theorists hold

by ganbayou Sun Feb 28, 2016 4:41 pm

I'm still not sure about B. Could anyone give another explanation to eliminate B? :(

Thank you
User avatar
 
tommywallach
Thanks Received: 468
Atticus Finch
Atticus Finch
 
Posts: 1041
Joined: August 11th, 2009
 
 
 

Re: Q15 - Bowers: A few theorists hold

by tommywallach Mon Feb 29, 2016 7:12 pm

There've been a few by now. I think if none of those work for you, just move on to another question! :)

-t
Tommy Wallach
Manhattan LSAT Instructor
twallach@manhattanprep.com
Image
 
ganbayou
Thanks Received: 0
Atticus Finch
Atticus Finch
 
Posts: 213
Joined: June 13th, 2015
 
 
 

Re: Q15 - Bowers: A few theorists hold

by ganbayou Wed Mar 02, 2016 8:27 pm

i thought it is time to review questions throughoutly... :(
User avatar
 
tommywallach
Thanks Received: 468
Atticus Finch
Atticus Finch
 
Posts: 1041
Joined: August 11th, 2009
 
 
 

Re: Q15 - Bowers: A few theorists hold

by tommywallach Mon Mar 07, 2016 10:08 pm

It is! But part of the work I do as forum manager is to consider when the threads get out of hand. Over time, the threads can fill up with the same question asked over and over, and answered over and over. At a certain point, I think this compromises clarity, rather than increases it. This question has been explained multiple times. No new explanation is likely to shed more light on it. I've perused the explanations already up, and I do think they are conclusive and persuasive. If they don't win you over, then it's really not worth worry about more! The question simply can't be explained any better. :)

-t
Tommy Wallach
Manhattan LSAT Instructor
twallach@manhattanprep.com
Image
 
WesleyC316
Thanks Received: 3
Jackie Chiles
Jackie Chiles
 
Posts: 40
Joined: March 19th, 2018
Location: Shanghai
 
 
 

Re: Q15 - Bowers: A few theorists hold

by WesleyC316 Fri May 25, 2018 7:46 am

On the third view, I'm actually getting more and more confused. I even made an analogous argument of my own to help myself see clearer.

Here's my argument:
A few maniacs hold the extreme view that restaurants can serve poops, people's faece. Some of these maniacs have even produced interesting arguments to support that position. One idiot, for example, contends that poop is distasteful meat taken to its logical extreme. But these maniacs' views ignore the fundamental principle of food philosophy, that an acceptable food must be edible. Any food that is not edible, i.e., poop, accordingly deserves no further attention.

My point is just that, I know there's a term shifting going on, but do we really need to justify that? If things so obvious like anarchy and poop need to be explained and justified, wouldn't LSATs be twice as long as it is now because they need to add tons of explanations just to make things crystal clear? Also I'm thinking, is term shifting always an error? When can we make term shifts then? And is it still a term shift if the author simply added "Oh by the way, anarchy countenances chaos. Although it's a fact, I'm still stating it, just in case some test writers put my argument in some tests and accuse me of shifting terms." in his argument?

I know these are probably irrelevant to the test, but I would be really appreciated if someone could discuss them with me!
 
bobjon1259
Thanks Received: 0
Vinny Gambini
Vinny Gambini
 
Posts: 16
Joined: November 27th, 2017
 
 
 

Re: Q15 - Bowers: A few theorists hold

by bobjon1259 Sat Jun 09, 2018 7:43 pm

.
Last edited by bobjon1259 on Mon Jan 22, 2024 12:18 am, edited 1 time in total.